Ultimo simulacro

Foro de discusion Sobre RFH

Moderador: Alberto

Avatar de Usuario
Bauer
Rn
Rn
Mensajes: 916
Registrado: 21 Feb 2007, 23:27

Ultimo simulacro

Mensaje por Bauer »

  • Los que no hayais realizado aun el simulacro no leais esto.

    Que os estoy avisando

    Hacerme caso, sobre todo tu, Alonso, que te conozco.

    Bueno, pues vosotros vereis

Otro simulacro, y otra cara de pardillo se me ha quedado (queria poner una palabrota, pero Acalon no me deja)

5) No puede ser la 2, ya que la aceleracion tangencial depende del radio. Yo puese la 4, entendiendo exterior del disco como todos los puntos situados a la mayor distancia posible

7) Relativo al centro del disco la velocidad será v=w*r, pero relativa a la superficie el radio, es decir, la distancia de giro, es doble y por tanto la velocidad también. A no ser que cambie w

9) El fetichismo de Acalon por los M.A.S. es increible. Si Epotencial=1/2*k*x^2 y Etotal = 1/2k*A^2, para A=4 y x=2 la relacion es 2^2/4^2 = 1/4

19) No tengo ni idea de como se hace, pero creo que que falta algun logaritmo neperiano

20, 21 y 22) Estas deberian anularse por ser demasiado largas. Odio los choques

24) Si en el fluido no hay rozamiento es como si fuera el vacio, por tanto la 2 y la 4 son ciertas. Vale, en un fluido hay una perdida de peso igual al peso del fluido desalojado, por lo que lo que la aceleracion depende del volumen y densidades. Anulada por cabrona (espero que Acalon me deje decir esta palabrota)

26) 5 MINUTOS PARA LA PREGUNTA Y LA RESPUESTA LA PONGO AL REVES, QUE GIL.....AS QUE SOY

27 y 28 ) Para estas cosas yo necesito la masa

33) La correcta es la 1. La Ec es maxima cuando x=0 y como kx=ma la aceleracion es cero. La 3 no puede ser

34) La 1 no es correcta, la 3 si

42) Si el objeto se mueve 0.5 m la imagen también, a no ser que pregunten por la distancia entre las dos nuevas posiciones, LO CUAL NO HACEN

52) Segun los angulos que te dan el indice es menor que 1. Ale, a la hoguera de las impugnaciones

Voy a seguir tirandome de los pelos. En breve pondré las del intervalo 50-100
Avatar de Usuario
Alonso
Xe
Xe
Mensajes: 535
Registrado: 02 May 2007, 22:12

Mensaje por Alonso »

Tío Bauer, tú no sabes que si quieres que la gente haga algo lo que debes hacer es decir que no lo hagan!

Nos veremos en el infierno Jack Bauer! :lol: :lol:

P.D. No he mirado las dudas.
Soy una onda, pero no se nota.
Avatar de Usuario
Patri
Cs
Cs
Mensajes: 540
Registrado: 20 Ene 2008, 22:13

Mensaje por Patri »

Hola chicos!!

Yo quería esperar aun un poco para meterme a preguntar los millones de dudas, pero sobre lo que has puesto, Bauer:

42_ Esta está bien. Lo que se mueve es el espejo. Se mueve 0,5m, así que la imagen se mueve el doble de espacio. Pero lo que se mueve es el espejo, no el objeto.

52_ A mí esta me da 0,909...

Estoy de acuerdo contigo en las demás, salvo en la 24. Ésta está bien. Tienes que considerar el peso y el empuje:
F=P-E=m*a.
Y de ahí, despejando la a, sale que no depende del volumen, pero sí de las densidades.

Creo que me pondré a corregirlo antes de lo que pensaba, porque la semana que viene con tanta fiesta...

Un saludo!!
Avatar de Usuario
Patri
Cs
Cs
Mensajes: 540
Registrado: 20 Ene 2008, 22:13

Mensaje por Patri »

Por cierto, me podrías explicar cómo haces la 26? Porque no me salió en el examen :cry: ...
Avatar de Usuario
Bauer
Rn
Rn
Mensajes: 916
Registrado: 21 Feb 2007, 23:27

Mensaje por Bauer »

La parte de optica y nuclear... aqui no he he estado tan "apardalao" como en las anteriores

55) Correcta la 1

57) No da ninguno exacto. 44.3º si suponemos que el angulo es respecto a la normal, o 56,4º si el angulo de 40º que dan es respecto al eje x, pero no puede ser el angulo menor si pasa de un medio a otro con menor indice

60) ¿Y por qué no puede ser la 1? Si el termino de superficie depende de A^2/3 cuanto mayor sea A, mayor será dicho término. Ademas, cuanto mayor sea A mas pequeño será el de asimetria ya que es inversamente proporcional. Y aun mas, lo dice Krane, pag 68, reduce su importancia para A grandes

67) Si depende fuertemente del spin, por algo llamdo muy pomposamente para mi gusto, acoplamiento spin-órbita invertido

73) Pues ahora que veo el resultado sí que me suena la pregunta y la respuesta, pero no lo veo...

87) La enesima vez que lo digo, es n=10
Avatar de Usuario
Patri
Cs
Cs
Mensajes: 540
Registrado: 20 Ene 2008, 22:13

Mensaje por Patri »

55_ De acuerdo.

57_ El ángulo que te dan es el de incidencia sobre el aceite. Tienes que hacer Snell 2 veces.

67_ De acuerdo.

73_No sé que decirte, no sé que es lo que no ves...

87_ Por enésima vez, de acuerdo :D .
Avatar de Usuario
Bauer
Rn
Rn
Mensajes: 916
Registrado: 21 Feb 2007, 23:27

Mensaje por Bauer »

Alonso, si no me haces caso lo pasarás mal :violent2:

42) Habia oido algo asi en el cole, cuando dicen "este" se refiere a lo ultimo que has dicho, o sea el espejo, y cuando dices "ese" te refieres a lo primero que dices, "objeto" en este caso. A repasar Lenguaje, señores

52) Es imposible que de mayor que 1 con los angulos que dan

24) Es cierto, no depende del volumen argggghhhh

26) Yo me acordaba de la parte sumergida de un iceberg = d_hielo/d_agua = 0.9, que yo puse en este problema que era la parte emergida, que cagada. Entonces d_objeto/ d_agua_salada y con los datos que dan sacas la densidad del objeto haciendo E= T + P

57) Si consideras que el rayo entre en el aceite desde el aire con ese angulo sale, pero el enunciado dice que brilla dentro del aceite con i=40º. Si con i quieren decir que es el angulo de incidencia del rayo que entra en el aceite desde el aire, pienso que es demasiada economia del lenguaje

73) No se si es un postulado o alguna movida cuantica o que. El problema de la cuantica es que pienso que es cosa de brujeria
Avatar de Usuario
Patri
Cs
Cs
Mensajes: 540
Registrado: 20 Ene 2008, 22:13

Mensaje por Patri »

Yo la 57 tampoco la hice bien a la primera, pero cuando veo que es un problema tan sencillo, de aplicar Snell sin más, y que no me sale, siempre pienso que puede haber gato encerrado en el enunciado. Se me ocurrió que podían estar refiriéndose a eso, lo probé, y me salió, así que no le dí más vueltas.
Avatar de Usuario
Bauer
Rn
Rn
Mensajes: 916
Registrado: 21 Feb 2007, 23:27

Mensaje por Bauer »

Mas de nuclear, que gusto da ver tantas respuestas en azul

116) Habia puesto la 1 en primera opcion, pero la cambié a la 5. Para un foton E=p*c = hc/L siendo L la longitud de onda --> L=h/p que es la longitud de onda Compton. Si p=m_e en valor, supongo, entonces L=7E-4 y no es 0,0243 Armstrongs. Falta, segun mi parecer, que el enunciad diga "momento igual a c veces la masa del electron"

139) ¿No tendria que ser la 2?

141) Puse la 4 sin saber muy bien por qué. He calculado el condensador en serie, he multiplicado por V y el resultado sale 4E-5. ¿No habria que dividir entre 2?

151) Si esta magnetizado, el polo norte del objeto será atraido por el S del iman, y viceversa, pero será repelido si ponemos los polos de la misma polaridad. La 1 y la 3 son ciertas, asi que queda la 5, que casualmente es la que puse

A comeeeeeeeer!
Avatar de Usuario
Patri
Cs
Cs
Mensajes: 540
Registrado: 20 Ene 2008, 22:13

Mensaje por Patri »

De acuerdo en todo lo que has escrito... Mis mismas dudas, mis mismas preguntas mal contestadas (o mal corregidas... yo no pierdo la esperanza :) )
Avatar de Usuario
et
Tb
Tb
Mensajes: 648
Registrado: 24 Nov 2005, 09:51

Mensaje por et »

ji ji lo acabo de terminar


ji ji ji ji ....................


Despues de comer miro las dudas




ji ji ji .........................
Avatar de Usuario
Bauer
Rn
Rn
Mensajes: 916
Registrado: 21 Feb 2007, 23:27

Mensaje por Bauer »

Ay Dios, que perdemos a la pingüina!!!

Lucha contra el simulacro!! Tu eres mas fuerte!!!
Avatar de Usuario
et
Tb
Tb
Mensajes: 648
Registrado: 24 Nov 2005, 09:51

Mensaje por et »

:occasion9: Hola Bauer!!

Toy tan cansada que solo he mirado las que tu dices y tambien estoy de acuerdo:

5) correcta la 4
7) y 9) idem la 1
27) y 28) yo tambien necesito la masa
33) y 34) idem la 1
52) anular
55) la 1
67) la 4
139) la 2

no se si hoy me dara el cuerpo pa mas

:sleepy2:
Avatar de Usuario
Patri
Cs
Cs
Mensajes: 540
Registrado: 20 Ene 2008, 22:13

Mensaje por Patri »

Bueno, pues aparte de corroborar todo lo que ha dicho hasta ahora Bauer, traigo mis dudas. Allá van:

11_ No sé cómo hacerla.
25_ Más de lo mismo.
36_ Y más...
40_ Ésta me sale la 2. La distancia que te dan es entre máximo y mínimo; así que: dist=lamda*dist.a.la.pant/(2*dist.entre.rend)
171_ ¿Por qué no vale la 4?
181_ ¿La 1 es falsa? Yo puse la 4, aunque sin mucha idea, la verdad...
183_ Ni idea.
195_ A mí ésta me sale la 4, que no hay solución.
201_ ¿El enunciado está bien? Por que a mí no me cuadra nada en esta pregunta...
202_ No sé cómo hacerla.
203_ Me sale que la distancia es 2; no sé si estoy haciendo algo mal o qué.
204_ Yo creía que sería 2/5. No entiendo qué tiene que ver lo que ocurra con la 2ª urna, con el hecho de que tu hayas sacado una blanca de la 1ª.
209_???
213_ Está mal.Es la 3.
223_ Aquí dice que faltan datos. ¿eso es porque no se puede suponer que es ideal? Porque muchas veces este examen va de suponer cosas a raiz de los enunciados que te dan...
244_ No sé de dónde sale eso.

Son un huevo, lo sé :oops:

Voy a seguir cortandome las venas...
Avatar de Usuario
Bauer
Rn
Rn
Mensajes: 916
Registrado: 21 Feb 2007, 23:27

Mensaje por Bauer »

Seguimos:

165) Con miniterminos creo que podria pero maxitérminos...

180) Estado solido y su prima la electrónica... yo diria que es la 4

181) Llamadme tiquismiquis, pero la exponencial es negativa, asi que la 2 es falsa

185) Yo creo que es la 3. Eso de que no se mueva...

196) Si a alguien le ha salido que son dependientes, me lo creeré, pero hasta que nadie lo diga son independientes, como mínimo, ya que no me puse a investigar si eran base o generadores, sobre todo por que ya no me acuerdo como se hacia eso

199) Veo porque no es continua en -1 pero no por que no es derivable en 3

202) Yo creo que es la 2, usease, que no existe, ya que no existe el limite del ln de x, cuando x tiende a cero por la izquierda

203) Sale una distancia de 2. A Acalon se le ha olvidado tener en cuente el 3 del otro lado del igual. Sacrificada al Dios de la Impugnaciones.

204 y 205) Se las dejo para el rey de la probabilidad, ichipiron

206) Me da una mediana de 3 y no de 4 como dice Acalon, ademas, ¿que le pasa a la moda cuando cuando hay dos frecuencias igual de altas? El tres y el cuatro se repiten 6 veces cada una

209) Claro, como me da para tanto un minuto, me da tiempo para girar esta figura y fumarme un canuto con los 23 segundos que me sobran.

212) Aqui se tiene en cuenta el númerito y da lo que dice Acalon

213) El vector (2, 1, -3) es perpendicular al plano. Si no fuera así, no podria haber hecho los problemas de distancias

221) La sal está para que traiga mala suerte si se vierte encima de la mesa. Yo creo que es la 2, aunque no hago mucho caso del tipo de preguntas con respuesta como "todo lo anterior hubiese dificultado mucho la tarea"

224) Vaya, en este tambien faltan datos. Si p*V /T = cte sale que es la octava parte del valor inicial

232) Si es indeformable, el W=0 --> U = Q. Es la 5

234 ) Alguien le tendria que decir a Acalon que en esta vida no hay nada perfecto. Yo creo que es la 1)

La 242 está en fase de análisis. Ya diré algo si alguna cosa no me cuadra

Uff, pensaba que nunca acabaria
Responder